Chapter 28: Lower Respiratory Problems

Ace your homework & exams now with Quizwiz!

Which of the following is the priority nursing intervention in helping a patient expectorate thick lung secretions? A. Humidify the oxygen as able B. Administer cough suppressant q4hr C. Teach patient to splint the affected area D. Increase fluid intake to 3 L/day if tolerated

d

The nurse reading a tuberculin skin test (TST) on a new employee who lives in the Midwest, is 20 years old, and has no known history of contact with any persons with tuberculosis (TB) will record it as positive if the area around the injection site has an area of swelling of _____ mm _____ hours after the injection. a. 15

48 b. 10; 72 c. 5; 48 d. 0 to 5; 72 ;ANS: A All TSTs are read at 48 to 72 hours after the injection. A positive reading of a TST for a person who is low risk for exposure is an area of swelling 15 mm or more. For individuals who are at high risk for TB (such as recent immigrants from countries where TB is prevalent, medically underserved groups, and the homeless), swelling of more than 10 mm is considered positive. Individuals with a history of contact with infectious TB or who are immunocompromised are considered to have a positive TST if there is more than 5 mm of swelling.

A 24-year-old male with a gunshot wound to the right side of the chest walks into the emergency department while leaning on another young man. The patient exhibits severe shortness of breath and decreased breath sounds on the right side. Which action should the nurse take immediately? A. Cover the chest wound with a nonporous dressing taped on three sides. B. Pack the chest wound with sterile saline soaked gauze and tape securely. C. Stabilize the chest wall with tape and initiate positive pressure ventilation. D. Apply a pressure dressing over the wound to prevent excessive loss of blood.

A The patient has a sucking chest wound (open pneumothorax). Air enters the pleural space through the chest wall during inspiration. Emergency treatment consists of covering the wound with an occlusive dressing that is secured on three sides. During inspiration the dressing pulls against the wound preventing air from entering the pleural space. During expiration the dressing is pushed out and air escapes through the wound and from under the dressing.

To promote airway clearance in a patient with pneumonia, the nurse instructs the patient to do which of the following (select all that apply)? A. Maintain adequate fluid intake B. Splint the chest when coughing C. Maintain a high Fowler's position D. Maintain a semi-Fowler's position E. Instruct patient to cough at end of exhalation

A, B, C, E

A 73-year-old female patient who lives alone is admitted to the hospital with a diagnosis of pneumococcal pneumonia. Which clinical manifestation, if observed by the nurse, indicates that the patient is likely to be hypoxic? A. Sudden onset of confusion B. Oral temperature of 102.3o F C. Coarse crackles in lung bases D. Clutching chest on inspiration

A. Confusion or stupor (related to hypoxia) may be the only clinical manifestation of pneumonia in an older adult patient. An elevated temperature, coarse crackles, and pleuritic chest pain with guarding may occur with pneumonia, but these symptoms do not indicate hypoxia.

The nurse notes new onset confusion in an older patient who is normally alert and oriented. In which order should the nurse take the following actions? (Put a comma and a space between each answer choice [A, B, C, D].) a. Obtain the oxygen saturation. b. Check the patient's pulse rate. c. Document the change in status. d. Notify the health care provider

ANS: A, B, D, C Assessment for physiologic causes of new onset confusion such as pneumonia, infection, or perfusion problems should be the first action by the nurse. Airway and oxygenation should be assessed first, then circulation. After assessing the patient, the nurse should notify the health care provider. Finally, documentation of the assessments and care should be done.

Two days after undergoing pelvic surgery, a patient develops marked dyspnea and anxiety. What is the first action that the nurse should take? a. Raise the head of the bed b. Notify the health care provider c. Take the patients pulse and blood pressure d. Determine the patients SpO2 with an oximeter

ANS: A All of the activities are correct but the first thing to do is to raise the head of the bed to promote respiration in the patient who is dyspneic. The health care provider would not be called until the nurse has assessment data relating to vital signs, pulse oximetry, and any other patient complains.

Which condition contributes to secondary pulmonary arterial hypertension by causing pulmonary capillary and alveolar damage? a. COPD b. Sarcoidosis c. Pulmonary fibrosis d. Pulmonary embolism

ANS: A Chronic obstructive pulmonary disease (COPD) causes pulmonary capillary and alveolar damage. Sarcoidosis is a granulomatous disease. Pulmonary fibrosis stiffens the pulmonary vasculature and pulmonary embolism obstructs pulmonary blood flow.

When obtaining a health history from a 76-year-old patient with suspected CAP, what does the nurse expect the patient or caregiver to report? a. Confusion b. A recent loss of consciousness c. An abrupt onset of fever and chills d. A gradual onset of headache and sore throat

ANS: A Confusion possibly related to hypoxia may be the only finding in older adults. Although CAP is most commonly caused by staphylococcus aureus pneumonia and is associated with an acute onset with fever, chills, productive cough with purulent or bloody sputum, and pleuritic chest pain, the older patient may not have classic symptoms. Other causes of pneumonia have a more gradual onset with dry, hacking cough; headache; and sore throat. A recent loss of consciousness or altered consciousness is common in those pneumonias associated with aspiration, such as anaerobic bacterial pneumonias.

Following assessment of a patient wit pneumonia, the nurse identifies a nursing diagnosis of impaired gas exchange based om which finding? a. SpO2 of 86% b. Crackles in both lobes c. Temperature of 101.4F (38.6C) d. Production of greenish purulent sputum

ANS: A Oxygen saturation obtained by pulse oximetry should be between 90% and 100%. An SpO2 lower than 90% indicates hypoxemia and impaired gas exchange. Crackles, purulent sputum, and fever are all manifestations of pneumonia, but do not necessarily related to impaired gas exchange.

The clinic nurse teaches a patient with a 42 pack-year history of cigarette smoking about lung disease. Which information will be most important for the nurse to include? a. Options for smoking cessation b. Reasons for annual sputum cytology testing c. Erlotinib (Tarceva) therapy to prevent tumor risk d. Computed tomography (CT) screening for lung cancer

ANS: A Because smoking is the major cause of lung cancer, the most important role for the nurse is teaching patients about the benefits of and means of smoking cessation. CT scanning is currently being investigated as a screening test for high-risk patients. However, if there is a positive finding, the person already has lung cancer. Erlotinib may be used in patients who have lung cancer, but it is not used to reduce the risk of developing cancer.

The nurse explains that a serious complication of a patient's chronic obstructive pulmonary disease (COPD) is cor pulmonale, which is exhibited by: a. distended neck veins. b. weight loss. c. confusion and disorientation. d. excessive coughing.

ANS: A Cor pulmonale is exhibited by distended neck veins, enlarged right side of the heart, liver engorgement, and edema.

The nurse administers prescribed therapies for a patient with cor pulmonale and right-sided heart failure. Which assessment would best evaluate the effectiveness of the therapies? a. Observe for distended neck veins. b. Auscultate for crackles in the lungs. c. Palpate for heaves or thrills over the heart. d. Review hemoglobin and hematocrit values.

ANS: A Cor pulmonale is right ventricular failure caused by pulmonary hypertension, so clinical manifestations of right ventricular failure such as peripheral edema, jugular venous distention, and right upper-quadrant abdominal tenderness would be expected. Crackles in the lungs are likely to be heard with left-sided heart failure. Findings in cor pulmonale include evidence of right ventricular hypertrophy on electrocardiogram ECG and an increase in intensity of the second heart sound. Heaves or thrills are not common with cor pulmonale. Chronic hypoxemia leads to polycythemia and increased total blood volume and viscosity of the blood. The hemoglobin and hematocrit values are more likely to be elevated with cor pulmonale than decreased.

A patient with bacterial pneumonia has rhonchi and thick sputum. What is the nurse's most appropriate action to promote airway clearance? a. Assist the patient to splint the chest when coughing. b. Teach the patient about the need for fluid restrictions. c. Encourage the patient to wear the nasal oxygen cannula. d. Instruct the patient on the pursed lip breathing technique.

ANS: A Coughing is less painful and more likely to be effective when the patient splints the chest during coughing. Fluids should be encouraged to help liquefy secretions. Nasal oxygen will improve gas exchange, but will not improve airway clearance. Pursed lip breathing is used to improve gas exchange in patients with COPD, but will not improve airway clearance.

When caring for a patient who is on a closed-chest drainage system with chest tubes, the nurse can confirm that the system is intact and working when: a. the water level in the water-seal chamber fluctuates. b. the level of fluid in the collection chamber rises. c. there are constant bubbles in the water-seal chamber. d. the suction has been attached.

ANS: A If the level of the water in the water-seal chamber rises and falls with the patient's respiration, the system is intact. Constant bubbles in the water-seal chamber indicate a leak in the system. The fluid in the collection container drains by gravity whether the closed-chest drainage system is intact or not. Suction is not significant with respect to whether the system is intact.

The nurse assesses the chest of a patient with pneumococcal pneumonia. Which finding would the nurse expect? a. Increased tactile fremitus b. Dry, nonproductive cough c. Hyperresonance to percussion d. A grating sound on auscultation

ANS: A Increased tactile fremitus over the area of pulmonary consolidation is expected with bacterial pneumonias. Dullness to percussion would be expected. Pneumococcal pneumonia typically presents with a loose, productive cough. Adventitious breath sounds such as crackles and wheezes are typical. A grating sound is more representative of a pleural friction rub rather than pneumonia.

The nurse schedules the postural drainage treatments to be done before breakfast because: a. fluids that have accumulated overnight can be expelled. b. bronchial openings are still more fully open after a night's rest. c. appetite will be stimulated for a meal after fluid is expelled. d. the empty stomach reduces gagging.

ANS: A Morning postural drainage can expel the fluids collected overnight. After a postural drainage, the patient is tired and may not want to eat at all. The empty stomach prevents excessive vomiting but not gagging.

An older patient is receiving standard multidrug therapy for tuberculosis (TB). The nurse should notify the health care provider if the patient exhibits which finding? a. Yellow-tinged skin b. Orange-colored sputum c. Thickening of the fingernails d. Difficulty hearing high-pitched voices

ANS: A Noninfectious hepatitis is a toxic effect of isoniazid (INH), rifampin, and pyrazinamide, and patients who develop hepatotoxicity will need to use other medications. Changes in hearing and nail thickening are not expected with the four medications used for initial TB drug therapy. Presbycusis is an expected finding in the older adult patient. Orange discoloration of body fluids is an expected side effect of rifampin and not an indication to call the health care provider.

The nurse cares for a patient who has just had a thoracentesis. Which assessment information obtained by the nurse is a priority to communicate to the health care provider? a. Oxygen saturation is 88%. b. Blood pressure is 145/90 mm Hg. c. Respiratory rate is 22 breaths/minute when lying flat. d. Pain level is 5 (on 0 to 10 scale) with a deep breath.

ANS: A Oxygen saturation would be expected to improve after a thoracentesis. A saturation of 88% indicates that a complication such as pneumothorax may be occurring. The other assessment data also indicate a need for ongoing assessment or intervention, but the low oxygen saturation is the priority.

A patient experiences a chest wall contusion as a result of being struck in the chest with a baseball bat. The emergency department nurse would be most concerned if which finding is observed during the initial assessment? a. Paradoxic chest movement b. Complaint of chest wall pain c. Heart rate of 110 beats/minute d. Large bruised area on the chest

ANS: A Paradoxic chest movement indicates that the patient may have flail chest, which can severely compromise gas exchange and can rapidly lead to hypoxemia. Chest wall pain, a slightly elevated pulse rate, and chest bruising all require further assessment or intervention, but the priority concern is poor gas exchange.

The patient with sleep apnea is fitted with a continuous positive airway pressure (CPAP) mask and asks the nurse how this device will help. The nurse correctly responds with which statement? a. "The device delivers constant positive pressure to keep your airway open." b. "The device will require you to be intubated to open your airway." c. "The device delivers oxygen only when you are apneic." d. "The device delivers negative pressure to stimulate your respirations."

ANS: A The CPAP mask delivers a constant positive pressure to keep the airway open. CPAP does not require intubation and does not deliver negative pressure.

A patient who was admitted the previous day with pneumonia complains of a sharp pain of 7 (based on 0 to 10 scale) "whenever I take a deep breath." Which action will the nurse take next? a. Auscultate breath sounds. b. Administer the PRN morphine. c. Have the patient cough forcefully. d. Notify the patient's health care provider.

ANS: A The patient's statement indicates that pleurisy or a pleural effusion may have developed and the nurse will need to listen for a pleural friction rub and/or decreased breath sounds. Assessment should occur before administration of pain medications. The patient is unlikely to be able to cough forcefully until pain medication has been administered. The nurse will want to obtain more assessment data before calling the health care provider

Following assessment of a patient with pneumonia, the nurse identifies a nursing diagnosis of ineffective airway clearance. Which assessment data best supports this diagnosis? a. Weak, nonproductive cough effort b. Large amounts of greenish sputum c. Respiratory rate of 28 breaths/minute d. Resting pulse oximetry (SpO2) of 85%

ANS: A The weak, nonproductive cough indicates that the patient is unable to clear the airway effectively. The other data would be used to support diagnoses such as impaired gas exchange and ineffective breathing pattern.

The nurse is caring for a patient who has a right-sided chest tube after a right lower lobectomy. Which nursing action can the nurse delegate to the unlicensed assistive personnel (UAP)? a. Document the amount of drainage every eight hours. b. Obtain samples of drainage for culture from the system. c. Assess patient pain level associated with the chest tube. d. Check the water-seal chamber for the correct fluid level.

ANS: A UAP education includes documentation of intake and output. The other actions are within the scope of practice and education of licensed nursing personnel.

What is the purpose of video-assissted thoracic surgery (VATS)? a. Removal of a lung b. Removal of one or more lung segments c. Removal of lung tissue by multiple wedge excision d. Used to inspect, diagnose, and manage intrathoractic injuries

ANS: D During video-assisted thoracic surgery (VATS), a video scope is inserted into the thorax to assess, diagnose, and treat intrathoractic injuries. A pneumonectomy is the removal of a lung. A wedge resection is the removal of a lung segment. Lung volume-reduction surgery is the removal of lung tissue by excising multiple wedges.

The nurse explains to the patient on a mechanical ventilator that it is set on assist-control mode, which means that the machine will: (Select all that apply.) a. deliver a set tidal volume. b. deliver a set number of breaths if the patient's rate falls. c. automatically cuts off if the patient is breathing independently. d. deliver more oxygen at the end of an inspiration. e. help to correct respiratory acidosis.

ANS: A, B The assist-control mode delivers a set tidal volume on every respiration and will deliver a set number of breaths per minute should the patient's rate drop. It does not cut off automatically or deliver more oxygen at the end of the inspiration, nor does it correct respiratory acidosis.

The nurse is working with a pulmonary specialist and is aware that the physician will most likely recommend that a full-year preventative protocol of isoniazid (INH) be given to people who: (Select all that apply.) a. are living with a person newly diagnosed as having tuberculosis. b. have had a positive tuberculin skin test but negative chest films. c. have had a positive tuberculin skin test and are on steroids. d. have had a positive tuberculin skin test and have diabetes. e. have had a positive tuberculin skin test and have had a gastrectomy.

ANS: A, B, C, D, E All options are people for whom a protocol of isoniazid should be recommended.

The home health nurse recommends to the 60-year-old patient with emphysema who is anorexic to enhance her nutrition by the practices of: (Select all that apply.) a. resting before eating. b. avoiding gas-producing food. c. eating four to six small meals rather than three large ones. d. lying down after eating. e. taking small bites and chewing slowly.

ANS: A, B, C, E Lying down after meals will likely increase shortness of breath. All other options will enhance her ability to increase her nutritional state.

The signs the nurse would expect to see in a patient with advanced emphysema are: (Select all that apply.) a. productive cough. b. dyspnea. c. barrel chest. d. wheezing. e. cyanotic skin tone.

ANS: A, B, C, E The emphysemic has a barrel chest and dyspnea. There is minimal coughing and mucus production until late in the disease. Wheezing usually does not occur in the emphysemic patient. Cyanosis is usually absent until late in the disease when the patient becomes hypoxic.

Which factors will the nurse consider when calculating the CURB-65 score for a patient with pneumonia (select all that apply)? a. Age b. Blood pressure c. Respiratory rate d. Oxygen saturation e. Presence of confusion f. Blood urea nitrogen (BUN) level

ANS: A, B, C, E, F Data collected for the CURB-65 are mental status (confusion), BUN (elevated), blood pressure (decreased), respiratory rate (increased), and age (65 and older). The other information is also essential to assess, but are not used for CURB-65 scoring.

Which medications would be used in four-drug treatment for the initial phase of TB(select all that apply)? a. Isoniazid (INH) b. Rifampin (Rifadin) c. Pyrazinamide (PZA) d. Rifabutin (Mycobutin) e. Levofloxacin (Levaquin) f. Ethambutol (Myambutol)

ANS: A, B, C, F For the first two months, a four-drug regimen consists of isoniazid (INH), rifampion (Rifadin), pyrazinamide (PZA), and ethambutol (Myambutol). Rifabutin ( Mycobutin) and levofloxacin (levaquin) may be used if the patient develops toxicity to the primary drugs. Rifabutin (Mycobutin) may be used as first-line treatment for patients receiving medications that interact with rifampin.

During a health promotion program, why should the nurse plan to target women in a discussion of lung cancer prevention (select all that apply)? a. Women develop lung cancer at a younger age than men b. More women die of lung cancer than die from breast cancer c. Women have a worse prognosis from lung cancer than do men d. women are more likely to develop small cell carcinoma than men t e. Nonsmoking women are at greater risk for developing lung cancer than men

ANS: A, B, D, E Smoking by women is taking a great toll, as reflected by the increasing incidence and deaths from lung cancer in women, who develop lung cancer at a younger age than men. Nonsmoking women are at greater risk of developing lung cancer. The incidence of small cell carcinoma is higher in women than in men> Men still have a worse prognosis than women from lung cancer.

The home health nurse making an initial call on a newly diagnosed tuberculosis patient who lives at home with his wife and child would give special instruction for infection control to: (Select all that apply.) a. place contaminated tissues in sealable plastic bag. b. take prescribed drug exactly as directed. c. take airborne precautions. d. wash hands frequently. e. wear mask when in crowds.

ANS: A, B, D, E As the family is already exposed, taking airborne precautions is unnecessary.

The nurse providing patient education states that influenza is spread by: (Select all that apply.) a. direct contact. b. indirect contact. c. vector. d. blood-borne method. e. droplets.

ANS: A, B, E Influenza is not spread by vectors or the blood-borne method.

Which of the following microorganisms are associated with both CAP and MCAP (select all that apply)? a. Kelbsiella b. Staphylococcus aureus c. Haemophilus influenzae d. Mycoplasma pneumonia e. Pseudomonas aeruginose f. Streptococcus pneumonia

ANS: A,B, F Community-acquired pneumonia (CAP) and medical care-associted pneumonia (MCAP) are both associated with Klebsiella, Streptococcus aureus, and Streptococcus pneumonia are only associated with CAP. Pseudomonas aeruginose is only associated with MCAP.

How do microorganisms reach the lungs and cause pneumonia (select all that apply)? a. Aspiration b. Lymphatic spread c. Inhalation of microbes in the air d. Touch contact with the infectious microbes e. Hematogenous spread from infections elsewhere in the body

ANS: A,C,E Microorganisms that cause pneumonia reach the lungs by aspiration from the nasopharynx or oropharynx, inhalation of microbes in the air, and hematogenous spread from infections elsewhere in the body. The other causes of infection do not contribute to pneumonia.

A patient diagnosed with class 3 TB 1 week ago is admitted to the hospital with syndrome of chest pain and coughing. What nursing action has the highest priority? a. Administering the patients anti tubercular drugs b. Admitting the patient to an airborne infection isolation room c. Preparing the patients room with suction equipment and extra linens d. Placing the patient in an intensive care unit where he can be closely monitored

ANS: B A patient with class 3 TB has clinically active disease and airborne infection isolation is required for active disease until the patient is noninfectious, indicated by negative sputum smears. Cardiac monitoring and observation will need to be done with the patient in isolation. The nurse will administer the anti tubercular drugs after the patient is in isolation. There should be no need for suction or extra linens after the TB patient is receiving drug therapy.

During an annual health assessment of a 65-year-old patient at the clinic, the patient tells the nurse he had the pneumonia vaccine when he was age 58. What should the nurse advise him about the best way for him to prevent pneumonia? a. Seek medical care and antibiotic therapy for all upper respiratory infections b. Obtain the pneumococcal vaccine this year with an annual influenza vaccine c. Obtain the pneumococcal vaccine if is he is exposed to individuals with pneumonia d. Obtain only the influenza vaccine every year because he has immunity to the pneumococcal

ANS: B A second dose of the pneumococcal vaccine should be provided to all persons 65 years of age or older who have not received the vaccine within 5 years and were younger than 65 years of age at the time of vaccination. Influenza vaccine should be taken each year by those older than 65 years of age. Antibiotic therapy is not appropriate for all upper respiratory infections unless secondary bacterial infections develop.

A pulmonary embolus is suspected in a patient with a deep vein thrombosis who develops hemoptysis, tachycardia, and chest pain.Diagnostic testing is scheduled. Which test should the nurse plan to teach the patient about? a. Chest x-rays b. Spiral (helical) CT scan c. Take the patients pulse and blood pressure d. Ventilation-perfusion lung scan

ANS: B A spiral (helical) CT scan is the most frequently used test to diagnosis pulmonary emboli because it allows illumination of all anatomic structures and produces a 3-D picture. If a patient cannot have contrast media, a ventilation-perfusion scan is done. Although pulmonary angiography is most sensitive, it is invasive, expensive, and carries more risk for complications. Chest x-rays do not detect pulmonary emboli until necrosis or abscesses occur.

To determine whether a tension pneumothorax is developing in a patient with chest trauma, for what does the nurse assess the patient? a. Dull percussion sounds on the injured side b. Severe respiratory distress and tracheal deviation c. Muffled and distant heart sounds with decreasing blood pressure d. Decreased movement and diminished breath sounds on the affected side

ANS: B A tension pneumothorax causes many of the same manifestations as other types of pneumothoraces but severe respiratory distress from collapse of the entire lung with movement of the mediastinal structures and trachea to the unaffected sides is present in a tension pneumothorax. Percussion dullness on the injured site indicates the presence of blood of fluid and decreased movement and diminished breath sounds are characteristic of a pneumothorax. Muffled and distant heart sounds indicate a cardiac tamponade.

To reduce the risk for most occupational lung diseases, what is the most important measure the occupational nurse should promote? a. Maintaining smoke-free work environments for all employees b. Using masks and effective ventilation systems to reduce exposure to irritants c. Inspection and monitoring of workplaces by national occupational safety agencies d. Requiring periodic chest x-rays and pulmonary function tests for exposed employees

ANS: B Although al of the precautions identified in this question are appropriate in decreasing the risk of occupational lung diseases, using masks and effective ventilation systems to reduce exposure is the most efficient and affects the greatest number of employees.

A patient with advanced lung cancer refuses pain medication, saying, "I deserve everything this cancer this cancer can give me." What is the nurse's best respond to this patient? a. "Would talking to a counselor help you?" b. "Can you tell me what the pain means to you/" c. "Are you using the pain as a punishment for your smoking/" d. "Pain control will help you to deal more effectively with your feelings."

ANS: B Before making any judgements about the patients statement, it is important to explore what meaning he or she finds in the pain. It may be that the patient feels it is deserved punishment for smoking but further information needs to b obtained from the patient. Immediate referral to a counselor negates the nurses responsibility in helping the patient and there is no indication that the patient is not dealing effectively with his or her feelings.

A patient with pneumonia has a nursing diagnosis of ineffective airway clearance related to pain, fatigue, and thick secretions. What is an expected outcome for this patient? a. SpO2 is 90% b. Lungs clear to auscultation c. Patient tolerates walking in hallway d. Patient take three or four shallow breaths before coughing to minimize pain

ANS: B Clear lung sounds indicate that the airways are clear. SpO2 of 90% to 100% indicates appropriate gas exchange. Tolerating walking in the hallway also indicates appropriate gas exchange, not approved airway clearance. Deep breaths are necessary to move mucus from distal airways but this is not an outcome.

Which chest surgery is used for the stripping of a fibrous membrane? a. Lobectomy b. Decortication c. Thoracotomy d. Wedge reaction

ANS: B Decortication is the stripping of a thick fibrous membrane. A lobectomy is the removal of one lung lobe. A thoracotomy is the incision into the thorax. A wedge resection is used to remove a small lesion.

What was the resurgence in tuberculosis (TB) resulting from the emergence of multidrug-resistant strains of Mycobacterium tuberculosis related to? a. A lack of effective means to diagnose TB b. Poor compliance with drug therapy in patients with TB c. Indiscriminate use of anti tubercular drugs in treatment of other infections d. Increased population of immunosuppressed individuals with acquired immunodeficiency syndrome (AIDS)

ANS: B Drug-resistant strains of TB have developed because TB patients compliance with drug therapy has been poor and there has been general decreased vigilance in monitoring and follow-up of TB treatment. TB can be diagnosed effectively with sputum cultures. Anti tubercular drugs are almost exclusively used for TB infections. The incidence of TB is at epidemic proportions in patients with HIV but this does not account for drug-resistant strains of TB.

While caring for a patient with idiopathic pulmonary arterial hypertension (IPAH), the nurse observes that the patient has exceptional dyspnea and chest pain in addition to fatigue. What are these symptoms related to? a. Decreased left ventricular output b. Right ventricular hypertrophy and dilation c. Increased systemic arterial blood pressure d. Development of alveolar interstitial edema

ANS: B High pressure in the pulmonary arteries increases the workload of the right ventricle and eventually causes right ventricular hypertrophy and dilation, known as cor pulmonale. Eventually, decreased left ventricular output may occur because of decreased return to the left atrium but it is not the primary effect of pulmonary hypertension. Alveolar interstitial edema is pulmonary edema associated with left ventricular failure. Pulmonary hypertension does not cause systemic hypertension.

The microorganisms Pneumocystis jiroveci (PCP) and cytomegalovirus (CMV) are associated with which type of pneumonia? a. Bronchial pneumonia b. Opportunistic pneumonia c. Hospital-associted pneumonia d. Community-acquired pneumonia

ANS: B People at risk for opportunistic pneumonia include those with altered immune response. Pneumocytisis jiroveci rarely causes pneumonia in persons with HIV disease. Cytomegalovirus (CMV) occurs in people with an impaired immune response. Medical care-associated pneumonia is frequently caused by Pseudomonas aeruginosa, Escherichia coli, Klebsiella, and Acinetobacter species. Community-acquired pneumonia is most commonly caused by streptococcus pneumonia.

63. When obtaining a health history from a patient suspected of having early TB, what manifestations should the nurse ask the patient about? a. Chest pain, hemoptysis, and weight loss b. Fatigue, low-grade fever, and night sweats c. Cough with purulent mucus and fever with chills d. Pleuritic pain, nonproductive cough, and temperature elevation at night

ANS: B TB usually develops insidiously with fatigue, malaise, anorexia, low-grade fevers, and night sweats. Pleuritic pain, flu-like symptoms, and a productive cough may occur with an acute sudden presentation but dyspnea and hemoptysis are late symptoms.

After 2 months of tuberculosis (TB) treatment with isoniazid (INH), rifampin (Rifadin), pyrazinamide (PZA), and ethambutol, a patient continues to have positive sputum smears for acid-fast bacilli (AFB). Which action should the nurse take next? a. Teach about treatment for drug-resistant TB treatment. b. Ask the patient whether medications have been taken as directed. c. Schedule the patient for directly observed therapy three times weekly. d. Discuss with the health care provider the need for the patient to use an injectable antibiotic.

ANS: B The first action should be to determine whether the patient has been compliant with drug therapy because negative sputum smears would be expected if the TB bacillus is susceptible to the medications and if the medications have been taken correctly. Assessment is the first step in the nursing process. Depending on whether the patient has been compliant or not, different medications or directly observed therapy may be indicated. The other options are interventions based on assumptions until an assessment has been completed.

When should the nurse check for leaks in the chest tube and pleural drainage system? a. There is continuous bubbling in the water-seal chamber b. There is constant bubbling of water in the suction control chamber c. Fluid in the water-seal chamber fluctuates with the patients breathing d. The water levels in the water-seal and suction control chambers are decreased

ANS: B The water-seal chamber should bubble intermittently as air leaves the lungs with exhalation in a spontaneously breathing patient. Continuos bubbling indicates a leak. The water in the suction control chamber will bubble continuously and the fluid in the water-seal chamber fluctuates with the patients breathing. Water in the suction control chamber, and perhaps in the water-seal chamber, evaporation and may need to be replaced periodically.

When caring for a patient who is hospitalized with active tuberculosis (TB), the nurse observes a student nurse who is assigned to take care of a patient. Which action, if performed by the student nurse, would require an intervention by the nurse? a. The patient is offered a tissue from the box at the bedside. b. A surgical face mask is applied before visiting the patient. c. A snack is brought to the patient from the unit refrigerator. d. Hand washing is performed before entering the patient's room.

ANS: B A high-efficiency particulate-absorbing (HEPA) mask, rather than a standard surgical mask, should be used when entering the patient's room because the HEPA mask can filter out 100% of small airborne particles. Hand washing before entering the patient's room is appropriate. Because anorexia and weight loss are frequent problems in patients with TB, bringing food to the patient is appropriate. The student nurse should perform hand washing after handling a tissue that the patient has used, but no precautions are necessary when giving the patient an unused tissue.

The 79-year-old patient with bacterial pneumonia becomes increasingly restless and confused. Temperature is 100° F and pulse, blood pressure, and respirations are elevated since the last assessment 6 hours ago. The initial intervention by the nurse should be to: a. take the patient off oral fluids. b. assess oxygen saturation. c. give the ordered mild sedative. d. administer an NSAID for discomfort.

ANS: B Assessing the oxygen saturation will reveal the level of oxygenation. These are early signs of hypoxia in the older adult. Medications for sedation or discomfort do not address the patient's current condition. There is no indication for stopping oral fluids.

The nurse is assessing the patient with influenza and notes general malaise and aching muscles, which have continued for 2 weeks. The nurse is aware that the patient may have developed which complication of influenza? a. Bronchitis b. Bacterial pneumonia c. Urinary infection d. Encephalitis

ANS: B Bacterial pneumonia is a common complication of influenza and may present with atypical symptoms of only general malaise and muscle aches, making it difficult to recognize the symptoms of pneumonia. Bronchitis, urinary infections, and encephalitis are not commonly complications of influenza.

A patient with idiopathic pulmonary arterial hypertension (IPAH) is receiving nifedipine (Procardia). Which assessment would best indicate to the nurse that the patient's condition is improving? a. Blood pressure (BP) is less than 140/90 mm Hg. b. Patient reports decreased exertional dyspnea. c. Heart rate is between 60 and 100 beats/minute. d. Patient's chest x-ray indicates clear lung fields.

ANS: B Because a major symptom of IPAH is exertional dyspnea, an improvement in this symptom would indicate that the medication was effective. Nifedipine will affect BP and heart rate, but these parameters would not be used to monitor the effectiveness of therapy for a patient with IPAH. The chest x-ray will show clear lung fields even if the therapy is not effective.

When the nurse assesses that the level of drainage has not increased over the last 3 hours on a first-day postoperative thoracotomy patient, the nurse should initially: a. inform the charge nurse. b. rearrange tubing to correct dependent loops. c. splint the patient with a pillow and coach to cough. d. gently massage the tube toward the collection bottle.

ANS: B Dependent loops can capture drainage and plug the tube, not allowing any more drainage to leave the chest. Massaging (milking) the tube can be implemented after the dependent loops have been corrected. It would not be necessary to notify the charge nurse unless there is a problem, and having the patient cough would not correct the drainage problem.

An experienced nurse instructs a new nurse about how to care for a patient with dyspnea caused by a pulmonary fungal infection. Which action by the new nurse indicates a need for further teaching? a. Listening to the patient's lung sounds several times during the shift b. Placing the patient on droplet precautions and in a private hospital room c. Increasing the oxygen flow rate to keep the oxygen saturation above 90% d. Monitoring patient serology results to identify the specific infecting organism

ANS: B Fungal infections are not transmitted from person to person. Therefore no isolation procedures are necessary. The other actions by the new nurse are appropriate

The 75-year-old resident in the nursing home who cares for 40 birds in an aviary complains of shortness of breath and fatigue and a dry cough. Based on this information, the nurse suspects the resident may be suffering from: a. coccidioidomycosis. b. histoplasmosis. c. tuberculosis. d. atypical pneumonia.

ANS: B Histoplasmosis is caused by a fungus that lives in bird droppings. The symptoms are dry cough, shortness of breath, and fatigue. Coccidioidomycosis is contracted by people who engage in desert recreational activities or are working in occupations that require digging in the earth. Tuberculosis and atypical pneumonia are not supported by the resident's history and symptoms.

A patient has just been admitted with probable bacterial pneumonia and sepsis. Which order should the nurse implement first? a. Chest x-ray via stretcher b. Blood cultures from two sites c. Ciprofloxacin (Cipro) 400 mg IV d. Acetaminophen (Tylenol) rectal suppository

ANS: B Initiating antibiotic therapy rapidly is essential, but it is important that the cultures be obtained before antibiotic administration. The chest x-ray and acetaminophen administration can be done last.

The nurse receives change-of-shift report on the following four patients. Which patient should the nurse assess first? a. A 23-year-old patient with cystic fibrosis who has pulmonary function testing scheduled b. A 46-year-old patient on bed rest who is complaining of sudden onset of shortness of breath c. A 77-year-old patient with tuberculosis (TB) who has four antitubercular medications due in 15 minutes d. A 35-year-old patient who was admitted the previous day with pneumonia and has a temperature of 100.2° F (37.8° C)

ANS: B Patients on bed rest who are immobile are at high risk for deep vein thrombosis (DVT). Sudden onset of shortness of breath in a patient with a DVT suggests a pulmonary embolism and requires immediate assessment and action such as oxygen administration. The other patients should also be assessed as soon as possible, but there is no indication that they may need immediate action to prevent clinical deterioration.

The nurse provides discharge instructions to a patient who was hospitalized for pneumonia. Which statement, if made by the patient, indicates a good understanding of the instructions? a. "I will call the doctor if I still feel tired after a week." b. "I will continue to do the deep breathing and coughing exercises at home." c. "I will schedule two appointments for the pneumonia and influenza vaccines." d. "I'll cancel my chest x-ray appointment if I'm feeling better in a couple weeks."

ANS: B Patients should continue to cough and deep breathe after discharge. Fatigue is expected for several weeks. The Pneumovax and influenza vaccines can be given at the same time in different arms. Explain that a follow-up chest x-ray needs to be done in 6 to 8 weeks to evaluate resolution of pneumonia

The nurse is teaching an asthma patient proper use of the peak flowmeter. The nurse determines further teaching is needed when observing which action by the patient? a. Repeating the procedure for a total of three readings b. Breathing in deeply through the mouthpiece c. Standing while performing the test d. Recording the highest reading on the peak flow sheet

ANS: B Peak flow should be monitored on a daily basis to determine if the asthma patient has adequate airflow. The reading helps determine if treatment should be adjusted. The patient should stand to achieve adequate chest expansion while taking a deep breath. The patient then blows as hard and fast as possible into the device with the mouthpiece in the mouth and the lips clamped firmly around it for a tight seal. The procedure should be performed three times with the highest reading recorded.

The patient with asthma is prescribed a leukotriene modifier drug, montelukast (Singulair). The nurse points out that the major advantage of this drug is it: a. has no GI side effects. b. provides bronchodilation and anti-inflammatory effects. c. controls acute asthma episodes. d. can be substituted for all other asthma remedies.

ANS: B Singulair provides both bronchodilation and anti-inflammatory effects, but it has numerous GI side effects and is not effective in controlling acute asthmatic attacks.

The nurse teaches a patient about the transmission of pulmonary tuberculosis (TB). Which statement, if made by the patient, indicates that teaching was effective? a. "I will avoid being outdoors whenever possible." b. "My husband will be sleeping in the guest bedroom." c. "I will take the bus instead of driving to visit my friends." d. "I will keep the windows closed at home to contain the germs."

ANS: B Teach the patient how to minimize exposure to close contacts and household members. Homes should be well ventilated, especially the areas where the infected person spends a lot of time. While still infectious, the patient should sleep alone, spend as much time as possible outdoors, and minimize time in congregate settings or on public transportation.

A patient is admitted to the emergency department with an open stab wound to the left chest. What is the first action that the nurse should take? a. Position the patient so that the left chest is dependent. b. Tape a nonporous dressing on three sides over the chest wound. c. Cover the sucking chest wound firmly with an occlusive dressing. d. Keep the head of the patient's bed at no more than 30 degrees elevation.

ANS: B The dressing taped on three sides will allow air to escape when intrapleural pressure increases during expiration, but it will prevent air from moving into the pleural space during inspiration. Placing the patient on the left side or covering the chest wound with an occlusive dressing will allow trapped air in the pleural space and cause tension pneumothorax. The head of the bed should be elevated to 30 to 45 degrees to facilitate breathing

The nurse supervises unlicensed assistive personnel (UAP) who are providing care for a patient with right lower lobe pneumonia. The nurse should intervene if which action by UAP is observed? a. UAP splint the patient's chest during coughing. b. UAP assist the patient to ambulate to the bathroom. c. UAP help the patient to a bedside chair for meals. d. UAP lower the head of the patient's bed to 15 degrees.

ANS: D Positioning the patient with the head of the bed lowered will decrease ventilation. The other actions are appropriate for a patient with pneumonia.

The nurse monitors a patient after chest tube placement for a hemopneumothorax. The nurse is most concerned if which assessment finding is observed? a. A large air leak in the water-seal chamber b. 400 mL of blood in the collection chamber c. Complaint of pain with each deep inspiration d. Subcutaneous emphysema at the insertion site

ANS: B The large amount of blood may indicate that the patient is in danger of developing hypovolemic shock. An air leak would be expected immediately after chest tube placement for a pneumothorax. Initially, brisk bubbling of air occurs in this chamber when a pneumothorax is evacuated. The pain should be treated but is not as urgent a concern as the possibility of continued hemorrhage. Subcutaneous emphysema should be monitored but is not unusual in a patient with pneumothorax. A small amount of subcutaneous air is harmless and will be reabsorbed.

A patient with newly diagnosed lung cancer tells the nurse, "I don't think I'm going to live to see my next birthday." Which response by the nurse is best? a. "Would you like to talk to the hospital chaplain about your feelings?" b. "Can you tell me what it is that makes you think you will die so soon?" c. "Are you afraid that the treatment for your cancer will not be effective?" d. "Do you think that taking an antidepressant medication would be helpful?"

ANS: B The nurse's initial response should be to collect more assessment data about the patient's statement. The answer beginning "Can you tell me what it is" is the most open-ended question and will offer the best opportunity for obtaining more data. The answer beginning, "Are you afraid" implies that the patient thinks that the cancer will be immediately fatal, although the patient's statement may not be related to the cancer diagnosis. The remaining two answers offer interventions that may be helpful to the patient, but more assessment is needed to determine whether these interventions are appropriate.

The nurse develops a plan of care to prevent aspiration in a high-risk patient. Which nursing action will be most effective? a. Turn and reposition immobile patients at least every 2 hours. b. Place patients with altered consciousness in side-lying positions. c. Monitor for respiratory symptoms in patients who are immunosuppressed. d. Insert nasogastric tube for feedings for patients with swallowing problems.

ANS: B The risk for aspiration is decreased when patients with a decreased level of consciousness are placed in a side-lying or upright position. Frequent turning prevents pooling of secretions in immobilized patients but will not decrease the risk for aspiration in patients at risk. Monitoring of parameters such as breath sounds and oxygen saturation will help detect pneumonia in immunocompromised patients, but it will not decrease the risk for aspiration. Conditions that increase the risk of aspiration include decreased level of consciousness (e.g., seizure, anesthesia, head injury, stroke, alcohol intake), difficulty swallowing, and nasogastric intubation with or without tube feeding. With loss of consciousness, the gag and cough reflexes are depressed, and aspiration is more likely to occur. Other high-risk groups are those who are seriously ill, have poor dentition, or are receiving acid-reducing medications.

A patient with a pleural effusion is scheduled for a thoracentesis. Which action should the nurse take to prepare the patient for the procedure? a. Start a peripheral IV line to administer the necessary sedative drugs. b. Position the patient sitting upright on the edge of the bed and leaning forward. c. Obtain a large collection device to hold 2 to 3 liters of pleural fluid at one time. d. Remove the water pitcher and remind the patient not to eat or drink anything for 6 hours.

ANS: B When the patient is sitting up, fluid accumulates in the pleural space at the lung bases and can more easily be located and removed. The patient does not usually require sedation for the procedure, and there are no restrictions on oral intake because the patient is not sedated or unconscious. Usually only 1000 to 1200 mL of pleural fluid is removed at one time. Rapid removal of a large volume can result in hypotension, hypoxemia, or pulmonary edema.

The nurse is preparing a presentation highlighting the benefits of annual influenza vaccination. The nurse correctly targets which groups? (Select all that apply.) a. The parents of children 3 to 6 months of age b. Diabetics who are over 50 years old c. Pregnant women d. Home health aides e. CNAs who work in long-term care facilities

ANS: B, C, D, E Children ages 6 to 59 months should receive the influenza vaccine, not children 3 to 6 months of age. The Advisory Committee on Immunization Practices also suggests that pregnant women, people over age 50, and people with certain chronic illnesses receive the vaccine. In addition, health care workers and those caring for persons in homes that are at high risk for contracting influenza should receive the vaccine.

After the health care provider sees a patient hospitalized with a stroke who developed a fever and adventitious lung sounds, the following orders are written. Which order should the nurse implement first? a. Anterior/posterior and lateral chest x-ray b. Start IV levofloxacin (Levaquin) 500 mg every 24 hrs c. Sputum specimen for Gram stain and culture and sensitivity d. Complete blood count (CBC) with white blood cell (WBC) count and differential

ANS: C A sputum specimen for gram stain and culture should be done before initiating antibiotic therapy and while waiting for the antibiotic to be delivered from the pharmacy in a hospitalized patient with suspected pneumonia and then antibiotics should be started without delay. If the sputum specimen cannot be obtained rapidly, the chest x-ray will be done to assess the typical pattern characteristic of the infecting organism. Blood cells tests will not be altered significantly by delaying the tests until after the first dose of antibiotics.

Six days after a heart-lung transplant, the patient develops a low-grade fever, dyspnea, and decreased SpO2. What should the nurse recognize that this may indicate? a. A normal response to extensive surgery b. A frequency fatal cytomegalovirus infection c. Acute rejection that may be treated with corticosteroids d. Obliterative bronchiolities that plugs terminal bronchioles

ANS: C Acute rejection may occur as early as 5 to 7 days after surgery and is manifested by low-grade fever, fatigue, and oxygen desaturation with exertion. Complete remission of symptoms can be accomplished with bolus corticosteroids. Cytomegalovirus and other infections can be fatal but usually occur weeks after surgery and manifest with symptoms of pneumonia. Obliterative bronchiolitis is a late complication of lung transplantation, reflecting chronic rejection.

A patient with a lung mass found on chest x-ray is undergoing further testing. The nurse explains that a diagnosis of lung cancer can be confirmed using which diagnostic test? a. Lung tomograms b. Pulmonary angiography c. Biopsy done via bronchoscopy d. Computed tomography (CT) scans

ANS: C Although chest x-rays, lung tomograms, CT scans, MRI, and positron emission tomography (PET) can identify tumors and masses, a definitive diagnosis of a lung cancer requires identification of malignant cells in either sputum specimens or biopsies.

An unlicensed assistive personnel (UAP) is taking care of a patient with a chest tube. The nurse should intervene when she observes the UAP? a. Looping the drainage tubing on the bed b.securing the drainage container in an upright position c. stripping or milking the chest tube to promote drainage d. reminding the patient to cough and deep breath every 2 hours

ANS: C If chest tubes are to be milked or stripped, this procedure should be done only by the professional nurse. This procedure is no longer recommended, as it may dangerously increase pleural pressure, but there is no indication to milk the tubes when there is no bloody drainage, as in a pneumothorax. The UAP can loop the chest tubing on the bed to promote drainage and patient s should be reminded to cough and deep breathe at least every 2 hours to aid in lung reexpansion. Securing the drainage container in an upright position is also a necessary activity that can be completed by UAP.

Why is the classification of pneumonia as community-acquired pneumonia (CAP) or medical care-associated pneumonia (MCAP) clinically useful? a. Atypical pneumonia syndrome is more likely to occur in MCAP b. Diagnostic testing does not have to be used to identify causative agents c. Causative agents can be predicted and empiric treatment is often effective d. IV antibiotics therapy is necessary for MCAP but oral therapy is adequate for CAP

ANS: C Pneumonia that has its onset in the community is usually caused by different microorganisms than pneumonia that develops related to hospitalization and treatment can be emoiric-based on observations and experience without knowing the exact causative organism. Frequently a causative organism cannot be identified from cultures and treatment is based on experience.

The nurse notes that a patient has incisional pain, a poor cough effort, and scattered rhonchi after a thoracotomy. Which action should the nurse take first? a. Assist the patient to sit upright in a chair. b. Splint the patient's chest during coughing. c. Medicate the patient with prescribed morphine. d. Observe the patient use the incentive spirometer.

ANS: C A major reason for atelectasis and poor airway clearance in patients after chest surgery is incisional pain (which increases with deep breathing and coughing). The first action by the nurse should be to medicate the patient to minimize incisional pain. The other actions are all appropriate ways to improve airway clearance but should be done after the morphine is given.

The nurse explains that treatment with amantadine (Symmetrel) will: a. prevent viral pneumonia if taken regularly. b. stop viral spread of avian flu if taken at the first signs and symptoms of disease. c. lessen the severity of type A flu symptoms if taken within 48 hours of exposure. d. reduce irritation of bronchitis if taken weekly.

ANS: C Amantadine (Symmetrel) is an antiviral medication that may be given within 48 hours of exposure or within 48 hours of the onset of influenza symptoms. It is not a drug that is taken regularly and will not stop the spread of the avian flu.

The patient with acute bronchitis asks if antibiotics will be ordered for the condition. The best response by the nurse would be: a. "Yes. Antibiotics are the best treatment option." b. "No. Antibiotics will not help a viral condition." c. "Antibiotics will be given if the sputum culture indicates your bronchitis is caused by bacteria." d. "I don't think so because antibiotics will inhibit the inflammatory response of your body to the invasion of this infection."

ANS: C Bronchitis is treated symptomatically with humidification and cough medications. Antibiotics are only given if the sputum culture suggests it.

A patient who has a right-sided chest tube following a thoracotomy has continuous bubbling in the suction-control chamber of the collection device. Which action by the nurse is most appropriate? a. Document the presence of a large air leak. b. Notify the surgeon of a possible pneumothorax. c. Take no further action with the collection device. d. Adjust the dial on the wall regulator to decrease suction.

ANS: C Continuous bubbling is expected in the suction-control chamber and indicates that the suction-control chamber is connected to suction. An air leak would be detected in the water-seal chamber. There is no evidence of pneumothorax. Increasing or decreasing the vacuum source will not adjust the suction pressure. The amount of suction applied is regulated by the amount of water in this chamber and not by the amount of suction applied to the system.

A patient has acute bronchitis with a nonproductive cough and wheezes. Which topic should the nurse plan to include in the teaching plan? a. Purpose of antibiotic therapy b. Ways to limit oral fluid intake c. Appropriate use of cough suppressants d. Safety concerns with home oxygen therapy

ANS: C Cough suppressants are frequently prescribed for acute bronchitis. Because most acute bronchitis is viral in origin, antibiotics are not prescribed unless there are systemic symptoms. Fluid intake is encouraged. Home oxygen is not prescribed for acute bronchitis, although it may be used for chronic bronchitis.

A patient is diagnosed with both human immunodeficiency virus (HIV) and active tuberculosis (TB) disease. Which information obtained by the nurse is most important to communicate to the health care provider? a. The Mantoux test had an induration of 7 mm. b. The chest-x-ray showed infiltrates in the lower lobes. c. The patient is being treated with antiretrovirals for HIV infection. d. The patient has a cough that is productive of blood-tinged mucus.

ANS: C Drug interactions can occur between the antiretrovirals used to treat HIV infection and the medications used to treat TB. The other data are expected in a patient with HIV and TB.

The nurse provides preoperative instruction for a patient scheduled for a left pneumonectomy for cancer of the lung. Which information should the nurse include about the patient's postoperative care? a. Positioning on the right side b. Bed rest for the first 24 hours c. Frequent use of an incentive spirometer d. Chest tube placement with continuous drainage

ANS: C Frequent deep breathing and coughing are needed after chest surgery to prevent atelectasis. To promote gas exchange, patients after pneumonectomy are positioned on the surgical side. Early mobilization decreases the risk for postoperative complications such as pneumonia and deep vein thrombosis. In a pneumonectomy, chest tubes may or may not be placed in the space from which the lung was removed. If a chest tube is used, it is clamped and only released by the surgeon to adjust the volume of serosanguineous fluid that will fill the space vacated by the lung. If the cavity overfills, it could compress the remaining lung and compromise the cardiovascular and pulmonary function. Daily chest x-rays can be used to assess the volume and space.

A patient who is taking rifampin (Rifadin) for tuberculosis calls the clinic and reports having orange discolored urine and tears. Which is the best response by the nurse? a. Ask if the patient is experiencing shortness of breath, hives, or itching. b. Ask the patient about any visual abnormalities such as red-green color discrimination. c. Explain that orange discolored urine and tears are normal while taking this medication. d. Advise the patient to stop the drug and report the symptoms to the health care provider.

ANS: C Orange-colored body secretions are a side effect of rifampin. The patient does not have to stop taking the medication. The findings are not indicative of an allergic reaction. Alterations in red-green color discrimination commonly occurs when taking ethambutol (Myambutol), which is a different TB medication.

An occupational health nurse works at a manufacturing plant where there is potential exposure to inhaled dust. Which action, if recommended by the nurse, will be most helpful in reducing the incidence of lung disease? a. Treat workers with pulmonary fibrosis. b. Teach about symptoms of lung disease. c. Require the use of protective equipment. d. Monitor workers for coughing and wheezing.

ANS: C Prevention of lung disease requires the use of appropriate protective equipment such as masks. The other actions will help in recognition or early treatment of lung disease but will not be effective in prevention of lung damage. Repeated exposure eventually results in diffuse pulmonary fibrosis. Fibrosis is the result of tissue repair after inflammation.

The health care provider writes an order for bacteriologic testing for a patient who has a positive tuberculosis skin test. Which action should the nurse take? a. Teach about the reason for the blood tests. b. Schedule an appointment for a chest x-ray. c. Teach about the need to get sputum specimens for 2 to 3 consecutive days. d. Instruct the patient to expectorate three specimens as soon as possible.

ANS: C Sputum specimens are obtained on 2 to 3 consecutive days for bacteriologic testing for M. tuberculosis. The patient should not provide all the specimens at once. Blood cultures are not used for tuberculosis testing. A chest x-ray is not bacteriologic testing. Although the findings on chest x-ray examination are important, it is not possible to make a diagnosis of TB solely based on chest x-ray findings because other diseases can mimic the appearance of TB.

The nurse is caring for a patient with idiopathic pulmonary arterial hypertension (IPAH) who is receiving epoprostenol (Flolan). Which assessment information requires the most immediate action by the nurse? a. The oxygen saturation is 94%. b. The blood pressure is 98/56 mm Hg. c. The patient's central IV line is disconnected. d. The international normalized ratio (INR) is prolonged.

ANS: C The half-life of this drug is 6 minutes, so the nurse will need to restart the infusion as soon as possible to prevent rapid clinical deterioration. The other data also indicate a need for ongoing monitoring or intervention, but the priority action is to reconnect the infusion.

Which action by the nurse will be most effective in decreasing the spread of pertussis in a community setting? a. Providing supportive care to patients diagnosed with pertussis b. Teaching family members about the need for careful hand washing c. Teaching patients about the need for adult pertussis immunizations d. Encouraging patients to complete the prescribed course of antibiotics

ANS: C The increased rate of pertussis in adults is thought to be due to decreasing immunity after childhood immunization. Immunization is the most effective method of protecting communities from infectious diseases. Hand washing should be taught, but pertussis is spread by droplets and contact with secretions. Supportive care does not shorten the course of the disease or the risk for transmission. Taking antibiotics as prescribed does assist with decreased transmission, but patients are likely to have already transmitted the disease by the time the diagnosis is made.

A patient with right lower-lobe pneumonia has been treated with IV antibiotics for 3 days. Which assessment data obtained by the nurse indicates that the treatment has been effective? a. Bronchial breath sounds are heard at the right base. b. The patient coughs up small amounts of green mucus. c. The patient's white blood cell (WBC) count is 9000/µL. d. Increased tactile fremitus is palpable over the right chest.

ANS: C The normal WBC count indicates that the antibiotics have been effective. All the other data suggest that a change in treatment is needed.

Employee health test results reveal a tuberculosis (TB) skin test of 16-mm induration and a negative chest x-ray for a staff nurse working on the pulmonary unit. The nurse has no symptoms of TB. Which information should the occupational health nurse plan to teach the staff nurse? a. Standard four-drug therapy for TB b. Need for annual repeat TB skin testing c. Use and side effects of isoniazid (INH) d. Bacille Calmette-Guérin (BCG) vaccine

ANS: C The nurse is considered to have a latent TB infection and should be treated with INH daily for 6 to 9 months. The four-drug therapy would be appropriate if the nurse had active TB. TB skin testing is not done for individuals who have already had a positive skin test. BCG vaccine is not used in the United States for TB and would not be helpful for this individual, who already has a TB infection.

The 30-year-old American Indian female who is taking Rifater, a drug containing rifampin, isoniazid, and pyrazinamide, complains that she is tired of taking medicine and having to spit in a bottle all the time. She asks, "When can I stop all this and get on with my life?" The nurse's best response is that she will no longer be considered contagious when: a. the sputum culture comes back negative. b. the medication has been taken for 9 months. c. three consecutive sputum cultures are negative. d. the tuberculin skin test (TST) is no longer positive.

ANS: C This drug is given to treat active tuberculosis. The active tuberculosis patient is considered noncontagious when three consecutive sputum cultures are negative. Taking the medication for a given period of time does not make the patient noncontagious. The TST will always be positive.

The nurse differentiates viral from bacterial pneumonia in that viral pneumonia causes: a. elevation in white count. b. consolidation of lung tissue. c. interstitial inflammation. d. copious exudate.

ANS: C Viral pneumonia causes interstitial inflammation with attendant edema. White count will not be elevated, and no exudate is consolidating the lung as with bacterial pneumonia.

Following a motor vehicle accident, the nurse assess the driver for which distinctive sign of flail chest? a. Severe hypotension b. Chest pain over ribs c. Absence of breath sounds d. Paradoxical chest movement

ANS: D Flail chest may occur when two or more ribs are fractured, causing an unstable segment. The chest wall cannot provide the support for ventilation and the injured segment will move paradoxically to the stable portion of the chest (in on exertion; out on inspiration). Hypotension occurs with a number of conditions that impair cardiac function, and chest pain occurs with a single fractured rib and will be of high priority with flail chest. Absent breath sounds occur following pneumothorax of hemothorax.

What is a primary treatment goal for cor pulmonale? a. Controlling dysrhythmias b. Dialing the pulmonary arteries c. Strengthening the cardiac muscle d. Treating the underlying pulmonary condition

ANS: D If possible, the primary management of for pulmonale is treatment of the underlying pulmonary problem that caused the heart problem. Low-flow oxygen therapy will help to prevent hypoxmia and hypercapnia, which causes pulmonary vasoconstriction.

A patient with active TB continues to have positive sputum cultures after 6 months of treatment. She says she cannot remember to take the medication all the time. What is the best action for the nurse to take? a. Schedule the patient to come to the clinic every day to take the medication b. Have a patient who has recovered from TB tell the patient about his successful treatment c. Schedule more teaching sessions so the patient will understand the risks of noncompliance d. Arrange for directly observed therapy by a responsible family member or a public health nurse

ANS: D Notification of the public health department is required. If drug compliance is questionable, follow-uo of patients can be made by directly observed therapy by a public health nurse or a responsible family member. A patient who cannot remember to take the medication usually will not remember to come to the clinic daily or will find it too inconvenient. Additional teaching or support from others is not usually effective for this type of patient.

What is the initial antibiotic treatment for pneumonia based on? a. The severity of symptoms b. The presence of characteristics leukocytes c. Gram stains and cultures of sputum specimens d. History and physical examination and characteristics chest x-ray findings

ANS: D Prompt treatment of pneumonia with appropriate antibiotics is important in treating bacterial and mycoplasma pneumonia and antibiotics are often administered on the basis of the history, physical examination, and a chest x-rays indicating a typical pattern characteristic of a particular organism without further testing. Sputum and blood cultures take 24 to 72 for results and microorganisms often cannot be identified with either Gram stains or cultures. Whether the pneumonia is community acquired or medical-care associated is more significant than the severity of symptoms.

A male patient has chronic obstructive pulmonary disease (COPD) and is a smoker. The nurse notices respiratory distress and no breath sounds over the left chest. Which type of pneumothorax should the nurse suspect is occurring? a. Tension pneumothorax b. Iatrogenic Pneumothorax c. Traumatic pneumothorax d. Spontaneous pneumothorax

ANS: D Spontaneous pneumothorax is seen from the rupture of small blebs on the apex of the lung in patents with lung disease or smoking, as well as in tall, thin males with a family history of or a previous spontaneous pneumothorax. Tension pneumothorax occurs with mechanical ventilation and with blocked check tubes. Iatrogenic pneumothorax occurs due to the laceration or puncture of the lung during medical procedures. Traumatic pneumothorax can occur with penetrating or blunt chest trauma.

A patient with a 40-year smoking history has recently stopped smoking because of the fear of developing lung cancer. The patient asks the nurse what can do to learn about whether he develops lung cancer. What is the best response from the nurse? a. "You should get a chest x-ray every 6 months to screen for any new growths." b. "It would be very rare for you to develop lung cancer now that you have stopped smoking." c. "You should monitor for any persistent cough, wheezing, or difficulty breathing, which could indicate tumor growth." d. "Screening measures for lung cancers are controversial, but we can discuss the advantages and disadvantages of various measures."

ANS: D The use of radiology, computed tomography (CT), and sputum cytology has been shown to detect lung cancer at earlier stages. Low-dose spiral CT scanning has been shown to decrease lung cancer mortality compared with those who has chest x-rays. To be considered for screening for lung cancer, patients must be 55 to 74 years old, current smokers with at least a 30 pack-year smoking history or former smokers who quit within the last 15 years, have no history of lung cancer, and not be on home oxygen. A patient who has a history of smoking always has an increased risk for lung cancer compared with an individual who has never smoked but the risk decreases as the period of nonsmoking increases.

Following a thoractomy, the patient has a nursing diagnosis of ineffective airway clearance related to inability to cough as a result of pain and positioning. What is the best nursing intervention for this patient? a. Have the patient drink 16 oz of water before attempting to deep breath b. Auscultate the lungs before and after deep-breathing and coughing regimens c. Place the patient in the Trendelenburg position for 30 minutes before the coughing exercise d. Medicate the patient with analgesics 20 to 30 minutes before assisting to cough and deep breathe.

ANS: D A thoracotomy incision is large and involves cutting into bone, muscle, and cartilage, resulting in significant postoperative pain. The patient has difficulty deep breathing and coughing because of the pain and analgesics should be provided before attempting these activities. Water intake is important to liquefy secretions but is not indicated in this case, nor should a patent with chest trauma or surgery be placed in Trendelenburg position, because it increases intrathoractic pressure.

A patient with pneumonia has a fever of 101.4° F (38.6° C), a nonproductive cough, and an oxygen saturation of 88%. The patient complains of weakness, fatigue, and needs assistance to get out of bed. Which nursing diagnosis should the nurse assign as the highest priority? a. Hyperthermia related to infectious illness b. Impaired transfer ability related to weakness c. Ineffective airway clearance related to thick secretions d. Impaired gas exchange related to respiratory congestion

ANS: D All these nursing diagnoses are appropriate for the patient, but the patient's oxygen saturation indicates that all body tissues are at risk for hypoxia unless the gas exchange is improved.

An alcoholic and homeless patient is diagnosed with active tuberculosis (TB). Which intervention by the nurse will be most effective in ensuring adherence with the treatment regimen? a. Arrange for a friend to administer the medication on schedule. b. Give the patient written instructions about how to take the medications. c. Teach the patient about the high risk for infecting others unless treatment is followed. d. Arrange for a daily noon meal at a community center where the drug will be administered.

ANS: D Directly observed therapy is the most effective means for ensuring compliance with the treatment regimen, and arranging a daily meal will help ensure that the patient is available to receive the medication. The other nursing interventions may be appropriate for some patients but are not likely to be as helpful for this patient.

A patient who has just been admitted with community-acquired pneumococcal pneumonia has a temperature of 101.6° F with a frequent cough and is complaining of severe pleuritic chest pain. Which prescribed medication should the nurse give first? a. Codeine b. Guaifenesin (Robitussin) c. Acetaminophen (Tylenol) d. Piperacillin/tazobactam (Zosyn)

ANS: D Early initiation of antibiotic therapy has been demonstrated to reduce mortality. The other medications are also appropriate and should be given as soon as possible, but the priority is to start antibiotic therapy.

Which intervention will the nurse include in the plan of care for a patient who is diagnosed with a lung abscess? a. Teach the patient to avoid the use of over-the-counter expectorants. b. Assist the patient with chest physiotherapy and postural drainage. c. Notify the health care provider immediately about any bloody or foul-smelling sputum. d. Teach about the need for prolonged antibiotic therapy after discharge from the hospital.

ANS: D Long-term antibiotic therapy is needed for effective eradication of the infecting organisms in lung abscess. Chest physiotherapy and postural drainage are not recommended for lung abscess because they may lead to spread of the infection. Foul smelling and bloody sputum are common clinical manifestations in lung abscess. Expectorants may be used because the patient is encouraged to cough

The nurse completes discharge teaching for a patient who has had a lung transplant. The nurse evaluates that the teaching has been effective if the patient makes which statement? a. "I will make an appointment to see the doctor every year." b. "I will stop taking the prednisone if I experience a dry cough." c. "I will not worry if I feel a little short of breath with exercise." d. "I will call the health care provider right away if I develop a fever."

ANS: D Low-grade fever may indicate infection or acute rejection so the patient should notify the health care provider immediately if the temperature is elevated. Patients require frequent follow-up visits with the transplant team. Annual health care provider visits would not be sufficient. Home oxygen use is not an expectation after lung transplant. Shortness of breath should be reported. Low-grade fever, fatigue, dyspnea, dry cough, and oxygen desaturation are signs of rejection. Immunosuppressive therapy, including prednisone, needs to be continued to prevent rejection.

A lobectomy is scheduled for a patient with stage I non-small cell lung cancer. The patient tells the nurse, "I would rather have chemotherapy than surgery." Which response by the nurse is most appropriate? a. "Are you afraid that the surgery will be very painful?" b. "Did you have bad experiences with previous surgeries?" c. "Surgery is the treatment of choice for stage I lung cancer." d. "Tell me what you know about the various treatments available."

ANS: D More assessment of the patient's concerns about surgery is indicated. An open-ended response will elicit the most information from the patient. The answer beginning, "Surgery is the treatment of choice" is accurate, but it discourages the patient from sharing concerns about surgery. The remaining two answers indicate that the nurse has jumped to conclusions about the patient's reasons for not wanting surgery. Chemotherapy is the primary treatment for small cell lung cancer. In non-small cell lung cancer, chemotherapy may be used in the treatment of nonresectable tumors or as adjuvant therapy to surgery

A patient is admitted with active tuberculosis (TB). The nurse should question a health care provider's order to discontinue airborne precautions unless which assessment finding is documented? a. Chest x-ray shows no upper lobe infiltrates. b. TB medications have been taken for 6 months. c. Mantoux testing shows an induration of 10 mm. d. Three sputum smears for acid-fast bacilli are negative.

ANS: D Negative sputum smears indicate that Mycobacterium tuberculosis is not present in the sputum, and the patient cannot transmit the bacteria by the airborne route. Chest x-rays are not used to determine whether treatment has been successful. Taking medications for 6 months is necessary, but the multidrug-resistant forms of the disease might not be eradicated after 6 months of therapy. Repeat Mantoux testing would not be done because the result will not change even with effective treatment.

The nurse is performing tuberculosis (TB) skin tests in a clinic that has many patients who have immigrated to the United States. Which question is most important for the nurse to ask before the skin test? a. "Is there any family history of TB?" b. "How long have you lived in the United States?" c. "Do you take any over-the-counter (OTC) medications?" d. "Have you received the bacille Calmette-Guérin (BCG) vaccine for TB?"

ANS: D Patients who have received the BCG vaccine will have a positive Mantoux test. Another method for screening (such as a chest x-ray) will need to be used in determining whether the patient has a TB infection. The other information also may be valuable but is not as pertinent to the decision about doing TB skin testing.

When caring for a patient with AIDS, the nurse is aware that this patient is most at risk for developing which type of pneumonia? a. Hypostatic b. Streptococcus pneumoniae c. Atypical d. Pneumocystis jiroveci

ANS: D Pneumocystis jiroveci (formerly known as Pneumocystis carinii) is commonly seen in AIDS patients. Hypostatic pneumonia is related to inadequate aeration of the lungs seen frequently with immobile patients. Streptococcus pneumoniae is the most common causative organism for bacterial pneumonia in the general population. Atypical pneumonia refers to pneumonia that does not present with the typical symptoms of pneumonia.

The 75-year-old patient asks the nurse if the Pneumovax immunization he took when he was 65 is still protecting him. The nurse's most helpful reply is: a. "Yes. Pneumovax protects you for your lifetime." b. "No. The immunity afforded you by Pneumovax lasts only 2 years." c. "Yes, but it loses strength and may not protect you from all 23 pneumococcal organisms anymore." d. "No. A second dose is needed 6 years after the first for full immunity."

ANS: D Pneumovax, an immunization that protects against 23 pneumococcal organisms, is repeated 6 years after the first dose.

A patient who has had a left pneumonectomy to remove a lung cancer is returned to the unit from surgery. The nurse should position the patient in a _____ position. a. high Fowler's b. semi-Fowler's c. right side-lying d. left side-lying

ANS: D Postoperative positioning after a pneumonectomy is on the operated side to prevent the threat of tension pneumothorax with mediastinal shift and leakage from the amputated bronchial stump. The physician's order should always be checked before turning the patient or raising the head of the bed.

The nurse provides discharge teaching for a patient who has two fractured ribs from an automobile accident. Which statement, if made by the patient, would indicate that teaching has been effective? a. "I am going to buy a rib binder to wear during the day." b. "I can take shallow breaths to prevent my chest from hurting." c. "I should plan on taking the pain pills only at bedtime so I can sleep." d. "I will use the incentive spirometer every hour or two during the day."

ANS: D Prevention of the complications of atelectasis and pneumonia is a priority after rib fracture. This can be ensured by deep breathing and coughing. Use of a rib binder, shallow breathing, and taking pain medications only at night are likely to result in atelectasis

A patient with a possible pulmonary embolism complains of chest pain and difficulty breathing. The nurse finds a heart rate of 142 beats/minute, blood pressure of 100/60 mmHg, and respirations of 42 breaths/minute. Which action should the nurse take first? a. Administer anticoagulant drug therapy. b. Notify the patient's health care provider. c. Prepare patient for a spiral computed tomography (CT). d. Elevate the head of the bed to a semi-Fowler's position.

ANS: D The patient has symptoms consistent with a pulmonary embolism (PE). Elevating the head of the bed will improve ventilation and gas exchange. The other actions can be accomplished after the head is elevated (and oxygen is started). A spiral CT may be ordered by the health care provider to identify PE. Anticoagulants may be ordered after confirmation of the diagnosis of PE.

An hour after a thoracotomy, a patient complains of incisional pain at a level 7 (based on 0 to 10 scale) and has decreased left-sided breath sounds. The pleural drainage system has 100 mL of bloody drainage and a large air leak. Which action is best for the nurse to take next? a. Milk the chest tube gently to remove any clots. b. Clamp the chest tube momentarily to check for the origin of the air leak. c. Assist the patient to deep breathe, cough, and use the incentive spirometer. d. Set up the patient controlled analgesia (PCA) and administer the loading dose of morphine.

ANS: D The patient is unlikely to take deep breaths or cough until the pain level is lower. A chest tube output of 100 mL is not unusual in the first hour after thoracotomy and would not require milking of the chest tube. An air leak is expected in the initial postoperative period after thoracotomy.

When assessing a patient who has just arrived after an automobile accident, the emergency department nurse notes tachycardia and absent breath sounds over the right lung. For which intervention will the nurse prepare the patient? a. Emergency pericardiocentesis b. Stabilization of the chest wall with tape c. Administration of an inhaled bronchodilator d. Insertion of a chest tube with a chest drainage system

ANS: D The patient's history and absent breath sounds suggest a right-sided pneumothorax or hemothorax, which will require treatment with a chest tube and drainage. The other therapies would be appropriate for an acute asthma attack, flail chest, or cardiac tamponade, but the patient's clinical manifestations are not consistent with these problems

After change-of-shift report, which patient should the nurse assess first? a. 72-year-old with cor pulmonale who has 4+ bilateral edema in his legs and feet b. 28-year-old with a history of a lung transplant and a temperature of 101° F (38.3° C) c. 40-year-old with a pleural effusion who is complaining of severe stabbing chest pain d. 64-year-old with lung cancer and tracheal deviation after subclavian catheter insertion

ANS: D The patient's history and symptoms suggest possible tension pneumothorax, a medical emergency. The other patients also require assessment as soon as possible, but tension pneumothorax will require immediate treatment to avoid death from inadequate cardiac output or hypoxemia.

The patient with chronic airflow limitation (CAL) changed to the use of a simple face mask from a nasal cannula delivery system. The nasal equipment oxygen was set at 3 L/min. The nurse instructs the patient that, with the change in delivery systems, the oxygen should be _____ L/min. a. decreased to 2 b. decreased to 1 c. increased to 4 d. increased to 6

ANS: D When changing to a mask from a nasal cannula, the oxygen should be increased by approximately 100% to get the same concentration. Simple face masks deliver approximately the same range of concentration of oxygen as the nasal cannula. However, the nasal cannula flow rates range from 1 L to 6 L, delivering 24% to 44% oxygen, whereas the simple face mask delivers 35% to 50% oxygen which is achieved with flow rates from 6 L to 12 L.

The nurse instructs a 38-year-old female patient with a pulmonary embolism about administering enoxaparin (Lovenox) after discharge. Which statement by the patient indicates understanding about the instructions? A. "I need to take this medicine with meals." B. "The medicine will be prescribed for 10 days." C. "I will inject this medicine into my upper arm." D. "The medicine will dissolve the clot in my lung."

B Enoxaparin is a low-molecular-weight heparin that is administered for 10 to 14 days and prevents future clotting but does not dissolve existing clots. Fibrinolytic agents (e.g., tissue plasminogen activator or alteplase) dissolve an existing clot. Enoxaparin is administered subcutaneously by injection into the abdomen.

The nurse cares for a 50-year-old patient with pneumonia that has been unresponsive to two different antibiotics. Which task is most important for the nurse to complete before administering a newly prescribed antibiotic? A. Teach the patient to cough and deep breathe. B. Take the temperature, pulse, and respiratory rate. C. Obtain a sputum specimen for culture and Gram stain. D. Check the patient's oxygen saturation by pulse oximetry.

C A sputum specimen for culture and Gram stain to identify the organism should be obtained before beginning antibiotic therapy. However, antibiotic administration should not be delayed if a specimen cannot be readily obtained because delays in antibiotic therapy can increase morbidity and mortality risks.

Which patient is at highest risk of aspiration? A. A 58-year-old patient with absent bowel sounds 12 hours after abdominal surgery B. A 67-year-old patient who had a cerebrovascular accident with expressive dysphasia C. A 26-year-old patient with continuous enteral tube feedings through a nasogastric tube D. A 52-year-old patient with viral pneumonia and coarse crackles throughout the lung fields

C. Conditions that increase the risk of aspiration include decreased level of consciousness, difficulty swallowing (dysphagia), and nasogastric intubation with or without tube feeding. With loss of consciousness, the gag and cough reflexes are depressed, and aspiration is more likely to occur. Dysphasia is difficulty with speech. Absent bowel sounds and coarse crackles do not increase the risk for aspiration.

Place the events of an asthma attack in proper sequence. a. Mast cell-mediated inflammatory response in bronchi b. Mucus production c. Plugging of small airways d. Contact with precipitator e. Mucosal edema

Step 1: d. Contact with precipitator Step 2: a. Mast cell-mediated inflammatory response in bronchi Step 3: e. Mucosal edema Step 4: b. Mucus production Step 5: c. Plugging of small airways

An appropriate nursing intervention for a pt with pneumonia with the nursing diagnosis of ineffective airway clearance related to thick secretions and fatigue would be to a. perform postural drainage every hour b. provide analgesics as ordered to promote pt comfort c. administer oxygen as prescribed to maintain optimal oxygen levels d. teach the pt how to cough effectively to bring secretions to the mouth

d

Guillain Barre syndrome causes respiratory problems primarily by a. depressing the CNS b. deforming chest wall muscles c. paralyzing the diaphragm secondary to trauma d. interrupting nerve transmission to respiratory muscles

d

A common complication of many types of environmental lung diseases is a. pulmonary fibrosis b. liquefactive necrosis c. benigh tumor growth d. diffuse airway obstruction

a

The nurse is caring for a patient admitted to the hospital with pneumonia. Upon assessment, the nurse notes a temperature of 101.4° F, a productive cough with yellow sputum, and a respiratory rate of 20. Which of the following nursing diagnosis is most appropriate based upon this assessment? A. Hyperthermia related to infectious illness B. Ineffective thermoregulation related to chilling C. Ineffective breathing pattern related to pneumonia D. Ineffective airway clearance related to thick secretions

a

Which of the following physical assessment findings in a patient with a lower respiratory problem best supports the nursing diagnosis of ineffective airway clearance? A. Basilar crackles B. Respiratory rate of 28 C. Oxygen saturation of 85% D. Presence of greenish sputum

a

When the patient with a persisting cough is diagnosed with pertussis (instead of acute bronchitis), the nurse knows that treatment will include which type of medication? a) Antibiotic b) Corticosteroid c) Bronchodilator d) Cough suppressant

a) Antibiotic Pertussis, unlike acute bronchitis, is caused by a gram-negative bacillus, Bordella pertussis, which must be treated with antibiotics. Corticosteroids and bronchodilators are not helpful in reducing symptoms. Cough suppressants and antihistamines are ineffective and may induce coughing episodes with pertussis.

Which physical assessment finding in a patient with a lower respiratory problem best supports the nursing diagnosis of ineffective airway clearance? a) Basilar crackles b) Respiratory rate of 28 c) Oxygen saturation of 85% d) Presence of greenish sputum

a) Basilar crackles The presence of adventitious breath sounds indicates that there is accumulation of secretions in the lower airways. This would be consistent with a nursing diagnosis of ineffective airway clearance because the patient is retaining secretions. The rapid respiratory rate, low oxygen saturation, and presence of greenish sputum may occur with a lower respiratory problem, but do not definitely support the nursing diagnosis of ineffective airway clearance.

The nurse is caring for a patient admitted to the hospital with pneumonia. Upon assessment, the nurse notes a temperature of 101.4° F, a productive cough with yellow sputum, and a respiratory rate of 20. Which nursing diagnosis is most appropriate based upon this assessment? a) Hyperthermia related to infectious illness b) Ineffective thermoregulation related to chilling c) Ineffective breathing pattern related to pneumonia d) Ineffective airway clearance related to thick secretions

a) Hyperthermia related to infectious illness Because the patient has spiked a temperature and has a diagnosis of pneumonia, the logical nursing diagnosis is hyperthermia related to infectious illness. There is no evidence of a chill, and her breathing pattern is within normal limits at 20 breaths/minute. There is no evidence of ineffective airway clearance from the information given because the patient is expectorating sputum.

During discharge teaching for a 65-year-old patient with chronic obstructive pulmonary disease (COPD) and pneumonia, which vaccine should the nurse recommend that this patient receive? a) Pneumococcal b) Staphylococcus aureus c) Haemophilus influenzae d) Bacille-Calmette-Guérin (BCG)

a) Pneumococcal The pneumococcal vaccine is important for patients with a history of heart or lung disease, recovering from a severe illness, age 65 or over, or living in a long-term care facility. A Staphylococcus aureus vaccine has been researched but not yet been effective. The Haemophilus influenzae vaccine would not be recommended as adults do not need it unless they are immunocompromised. The BCG vaccine is for infants in parts of the world where tuberculosis (TB) is prevalent.

During admission of a patient diagnosed with non-small cell lung carcinoma, the nurse questions the patient related to a history of which risk factors for this type of cancer (select all that apply)? a) Asbestos exposure b) Exposure to uranium c) Chronic interstitial fibrosis d) History of cigarette smoking e) Geographic area in which he was born

a, b, & d Non-small cell carcinoma is associated with cigarette smoking and exposure to environmental carcinogens, including asbestos and uranium. Chronic interstitial fibrosis is associated with the development of adenocarcinoma of the lung. Exposure to cancer-causing substances in the geographic area where the patient has lived for some time may be a risk, but not necessarily where the patient was born.

For which patients with pneumonia would the nurse suspect aspiration as the likely cause of pneumonia (select all that apply)? a. Patient with seizures b. Patient with head injury c. Patient who had thoracic surgery d. Patient who had a myocardial infarction e. Patient who is receiving nasogastric tube feeding

a, b, & e

To promote airway clearance in a patient with pneumonia, what should the nurse instruct the patient to do (select all that apply)? a) Maintain adequate fluid intake. b) Splint the chest when coughing. c) Maintain a 30-degree elevation. d) Maintain a semi-Fowler's position. e) Instruct patient to cough at end of exhalation.

a, b, & e Maintaining adequate fluid intake liquefies secretions, allowing easier expectoration. The nurse should instruct the patient to splint the chest while coughing. This will reduce discomfort and allow for a more effective cough. Coughing at the end of exhalation promotes a more effective cough. The patient should be positioned in an upright sitting position (high Fowler's) with head slightly flexed.

Which statement(s) describe(s) the management of a patient following lung transplantation (select all that apply)? a. The lung is biopsied using a transtracheal method b. High doses of oxygen are administered around the clock c. The use of a home spirometer will help to monitor lung function d. Immunosuppressant therapy usually involves a three-drug regimen e. Most patients experience an acute rejection episode in the first 3 days.

a, c, & d

Which of the following statements describe the management of a patient following lung transplantation (select all that apply) a. the lung is biopsied using a transtracheal method b. high doses of oxygen are administered around the clock c. the use of a home spirometer will help to monitor lung function d. immunosuppressant therapy usually involves a three drug regimen e. most pt's will experience an acute rejection episode in the first 3 days

a, c, d

When admitting a 45-year-old female with a diagnosis of pulmonary embolism, the nurse will assess the patient for which risk factors (select all that apply)? a) Obesity b) Pneumonia c) Malignancy d) Cigarette smoking e) Prolonged air travel

a, c, d, & e An increased risk of pulmonary embolism is associated with obesity, malignancy, heavy cigarette smoking, and prolonged air travel with reduced mobility. Other risk factors include deep vein thrombosis, immobilization, surgery within the last 3 months, oral contraceptives and hormone therapy, heart failure, pregnancy, and clotting disorders.

A priority nursing intervention for a patient who has just undergone a chemical pleurodesis for recurrent pleural effusion is: a. administering ordered analgesia b. monitoring chest tube drainage c. sending pleural fluid for laboratory analysis d. monitoring the patient's level of consciousness

a. administering ordered analgesia

When caring for a patient with acute bronchitis, the nurse will prioritize: a. auscultating lung sounds b. encouraging fluid restriction c. administering antibiotic therapy d. teaching the patient to avoid cough suppressants

a. auscultating lung sounds

A patient has been receiving high-dose corticosteroids and broad-spectrum antibiotics for treatment secondary to a traumatic injury and infection. The nurse plans care for the patient knowing that the patient is most susceptible to: a. candidiasis b. aspergillosis c. histoplasmosis d. coccidioidomycosis

a. candidiasis

The nurse notes tidaling of the water level in the tube submerged in the water-seal chamber in a patient with closed chest tube drainage. The nurse should: a. continue to monitor the patient b. check all connections for a leak in the system c. lower the drainage collector further from the chest d. clamp the tubing at progressively distal points away from the patient until the tidaling stops

a. continue to monitor the patient

The emergency department nurse is caring for patients exposed to a chlorine leak from a local factory. The nurse would closely monitor these patients for: a. pulmonary edema b. anaphylactic shock c. respiratory alkalosis d. acute tubular necrosis

a. pulmonary edema

After admitting a patient to the medical unit with a diagnosis of pneumonia, the nurse will verify that which of the following physician orders have been completed before administering a dose of cefotetan (Cefotan) to the patient? A. Orthostatic blood pressures B. Sputum culture and sensitivity C. Pulmonary function evaluation D. Serum laboratory studies ordered for am

b

Which statement best describes the treatment of lung abscess? a. it is best treated with surgical excision and drainage b. antibiotics given for a prolonged period are the usual treatment of choice c. the abscess is difficult to treat and frequently results in pulmonary fibrosis d. Penicillin can effectively eradicate the anaerobic organisms causing the abscess

b

After admitting a patient from home to the medical unit with a diagnosis of pneumonia, which physician orders will the nurse verify have been completed before administering a dose of cefuroxime (Ceftin) to the patient? a) Orthostatic blood pressures b) Sputum culture and sensitivity c) Pulmonary function evaluation d) Serum laboratory studies ordered for AM

b) Sputum culture and sensitivity The nurse should ensure that the sputum for culture and sensitivity was sent to the laboratory before administering the cefuroxime as this is community-acquired pneumonia. It is important that the organisms are correctly identified (by the culture) before the antibiotic takes effect. The test will also determine whether the proper antibiotic has been ordered (sensitivity testing). Although antibiotic administration should not be unduly delayed while waiting for the patient to expectorate sputum, orthostatic BP, pulmonary function evaluation, and serum laboratory tests will not be affected by the administration of antibiotics.

When planning appropriate nursing interventions for a patient with metastatic lung cancer and a 60-pack-per-year history of cigarette smoking, the nurse recognizes that the smoking has most likely decreased the patient's underlying respiratory defenses because of impairment of a) cough reflex. b) mucociliary clearance. c) reflex bronchoconstriction. d) ability to filter particles from the air.

b) mucociliary clearance. Smoking decreases the ciliary action in the tracheobronchial tree, resulting in impaired clearance of respiratory secretions and particles, chronic cough, and frequent respiratory infections.

In assessing a pt with pneumococcal pneumonia, the nurse recognizes that clinical manifestations of this condition include (select all that apply) a. nonproductive cough b. an abrupt onset of fever c. night sweats that are usually self limiting d. productive cough with rust colored sputum e. a gradual onset of sore throat and purulent productive cough.

b, d

When caring for a patient with a lung abscess, what is the nurse's priority intervention? a. Postural drainage b. Antibiotic administration c. Obtaining a sputum specimen d. Patient teaching regarding home care

b. Antibiotic administration

The nurse receives an order for a patient with lung cancer to receive influenza vaccine and pneumococcal vaccines. The nurse will: a. call the healthcare provider to question the order. b. administer both vaccines at the same time in different arms c. administer the flu shot and tell the patient to come back d. week later to receive the pneumococcal vaccine and suggest FluMist (nasal vaccine) instead of the influenza injection.

b. administer both vaccines at the same time in different arms

When planning care for a patient at risk for pulmonary embolism, the nurse prioritizes: a. maintaining the patient on bed rest b. using sequential compression devices c. encouraging the patient to cough and deep breathe d. teaching the patient how to use the incentive spirometer

b. using sequential compression devices

A nursing measure that should be instituted after a pneumonectomy is a. monitoring chest tube drainage and functioning b. positioning the pt on the unaffected side or back c. range of motion exercises on the affected upper extremity d. auscultating frequently for lung sounds on the affected side

c

A pt is on a continuous epoprostenol infusion pump. The alarm goes off indicating an obstruction in the intravenous line downstream. The nurse should a. check vital signs and oxygen saturation b. auscultate the lungs for pulmonary congestion c. assess the central line immediately for any obstruction or accidental clamping of tubing d. monitor for flushing and hypotension due to rebound from no medication and the short half life of the drug

c

During discharge teaching for a 65-year-old patient with COPD and pneumonia, which of the following vaccines should the nurse recommend that this patient receive? A. a. Staphylococcus aureus B. Haemophilus influenzae C. Pneumococcal D. Bacille-Calmette-Guérin (BCG)

c

The nurse evaluates that discharge teaching for a patient hospitalized with pneumonia has been most effective when the patient states which of the following measures to prevent a relapse? A. "I will seek immediate medical treatment for any upper respiratory infections." B. "I will increase my food intake to 2400 calories a day to keep my immune system well." C. "I should continue to do deep-breathing and coughing exercises for at least 6 weeks." D. "I must use home oxygen therapy for 3 months and then will have a chest x-ray to reevaluate."

c

The nurse is caring for a 73-year-old patient who underwent a left total knee arthroplasty. On the third postoperative day, the patient complains of shortness of breath, slight chest pain, and that "something is wrong." Temperature is 98.4o F, blood pressure 130/88, respirations 36, and oxygen saturation 91% on room air. Which of the following should the nurse first suspect as the etiology of this episode? A. New onset of angina pectoris B. Septic embolus from the knee joint C. Pulmonary embolus from deep vein thrombosis D. Pleural effusion related to positioning in the operating room

c

Which of the following clinical manifestations would the nurse expect to find during assessment of a patient admitted with pneumococcal pneumonia? A. Hyperresonance on percussion B. Vesicular breath sounds in all lobes C. Increased vocal fremitus on palpation D. Fine crackles in all lobes on auscultation

c

The patient with HIV has been diagnosed with Candida albicans, an opportunistic infection. The nurse knows the patient needs more teaching when she says, a) "I will be given amphotericin B to treat the fungus." b) "I got this fungus because I am immunocompromised." c) "I need to be isolated from my family and friends so they won't get it." d) "The effectiveness of my therapy can be monitored with fungal serology titers."

c) "I need to be isolated from my family and friends so they won't get it." The patient with an opportunistic fungal infection does not need to be isolated because it is not transmitted from person to person. This immunocompromised patient will be likely to have a serious infection so it will be treated with IV amphotericin B. The effectiveness of the therapy can be monitored with fungal serology titers.

One week after a thoracotomy, a patient with chest tubes (CTs) to water-seal drainage has an air leak into the closed chest drainage system (CDS). Which patient assessment warrants follow-up nursing interventions? a) Water-seal chamber has 5 cm of water. b) No new drainage in collection chamber c) Chest tube with a loose-fitting dressing d) Small pneumothorax at CT insertion site

c) Chest tube with a loose-fitting dressing If the dressing at the CT insertion site is loose, an air leak will occur and will need to be sealed. The water-seal chamber usually has 2 cm of water, but having more water will not contribute to an air leak, and it should not be drained from the CDS. No new drainage does not indicate an air leak but may indicate the CT is no longer needed. If there is a pneumothorax, the chest tube should remove the air.

Which clinical manifestation should the nurse expect to find during assessment of a patient admitted with pneumonia? a) Hyperresonance on percussion b) Vesicular breath sounds in all lobes c) Increased vocal fremitus on palpation d) Fine crackles in all lobes on auscultation

c) Increased vocal fremitus on palpation A typical physical examination finding for a patient with pneumonia is increased vocal fremitus on palpation. Other signs of pulmonary consolidation include bronchial breath sounds, egophony, and crackles in the affected area. With pleural effusion, there may be dullness to percussion over the affected area.

In the case scenario in question 15 above, which of the following actions should the nurse take first? A. Notify the physician. B. Administer a nitroglycerine tablet sublingually. C. Conduct a thorough assessment of the chest pain. D. Sit the patient up in bed as tolerated and apply oxygen.

d

The pt with lung cancer needs to receive influenza vaccine and pneumococcal vaccines. The nurse will a. administer both vaccines at the same time in the same arm b. administer both vaccines at the same time in different arms c. administer the flu shot and tell the pt to come back 1 week later to receive the pneumococcal vaccine d. administer the pneumococcal vaccine and suggest Flumist (nasal vaccine) instead of the influenza injection

d

While ambulating a patient with metastatic lung cancer, the nurse observes a drop in oxygen saturation from 93% to 86%. Which nursing intervention is most appropriate based upon these findings? a) Continue with ambulation since this is a normal response to activity. b) Obtain a physician's order for arterial blood gas determinations to verify the oxygen saturation. c) Obtain a physician's order for supplemental oxygen to be used during ambulation and other activity. d) Move the oximetry probe from the finger to the earlobe for more accurate monitoring during activity.

c) Obtain a physician's order for supplemental oxygen to be used during ambulation and other activity. An oxygen saturation level that drops below 90% with activity indicates that the patient is not tolerating the exercise and needs to use supplemental oxygen. The patient will need to rest to resaturate. ABGs or moving the probe will not be needed as the pulse oximeter was working at the beginning of the walk.

When the patient is diagnosed with a lung abscess, what should the nurse teach the patient? a) Lobectomy surgery is usually needed to drain the abscess. b) IV antibiotic therapy will be used for a prolonged period of time. c) Oral antibiotics will be used when the patient and x-ray shows evidence of improvement. d) No further culture and sensitivity tests are needed if the patient takes the medication as ordered.

c) Oral antibiotics will be used when the patient and x-ray shows evidence of improvement. IV antibiotics are used until the patient and x-ray show evidence of improvement. Then oral antibiotics are used for a prolonged period of time. Culture and sensitivity testing is done during the course of antibiotic therapy to ensure that the infecting organism is not becoming resistant to the antibiotic as well as at the completion of the antibiotic therapy. Lobectomy surgery is only needed when reinfection of a large cavitary lesion occurs or to establish a diagnosis when there is evidence of a neoplasm or other underlying problem.

What nursing intervention is most appropriate to enhance oxygenation in a patient with unilateral malignant lung disease? a) Positioning patient on right side b) Maintaining adequate fluid intake c) Positioning patient with "good lung" down d) Performing postural drainage every 4 hours

c) Positioning patient with "good lung" down Therapeutic positioning identifies the best position for the patient, thus assuring stable oxygenation status. Research indicates that positioning the patient with the unaffected lung (good lung) dependent best promotes oxygenation in patients with unilateral lung disease. For bilateral lung disease, the right lung down has best ventilation and perfusion. Increasing fluid intake and performing postural drainage will facilitate airway clearance, but positioning is most appropriate to enhance oxygenation.

A patient with TB has been admitted to the hospital and is placed in an airborne infection isolation room. What should the patient be taught (select all that apply)? a. Expect routine TST to evaluate infection. b. Visitors will not be allowed while in airborne isolation. c. Take all medication for full length of time to prevent multi drug-resistant TB. d. Wear a standard isolation mask if leaving the airborne infection isolation room. e. Maintain precautions in airborne infection isolation room by coughing into a paper tissue.

c, d, & e

An appropriate nursing intervention for a patient postpneumonectomy is: a. monitoring chest tube drainage and functioning b. positioning the patient on the unaffected side or back c. doing range-of-motion exercises on the affected upper limb d. auscultating frequently for lung sounds on the affected side

c. doing range-of-motion exercises on the affected upper limb

The nurse identifies a flail chest in a trauma patient when: a. multiple rib fractures are determined by x-ray b. a tracheal deviation to the unaffected side is present c. paradoxic chest movement occurs during respiration d. there is decreased movement of the involved chest wall

c. paradoxic chest movement occurs during respiration

The nurse evaluates that discharge teaching for a patient hospitalized with pneumonia has been effective when the patient makes which statement about measures to prevent a relapse? a) "I will seek immediate medical treatment for any upper respiratory infections." b) "I should continue to do deep-breathing and coughing exercises for at least 12 weeks." c) "I will increase my food intake to 2400 calories a day to keep my immune system well." d) "I must have a follow-up chest x-ray in 6 to 8 weeks to evaluate the pneumonia's resolution."

d) "I must have a follow-up chest x-ray in 6 to 8 weeks to evaluate the pneumonia's resolution." The follow-up chest x-ray will be done in 6 to 8 weeks to evaluate pneumonia resolution. A patient should seek medical treatment for upper respiratory infections that persist for more than 7 days. It may be important for the patient to continue with coughing and deep breathing exercises for 6 to 8 weeks, not 12 weeks, until all of the infection has cleared from the lungs. Increased fluid intake, not caloric intake, is required to liquefy secretions.

The patient who had idiopathic pulmonary fibrosis had a bilateral lung transplantation. Now he is experiencing airflow obstruction that is progressing over time. It started with a gradual onset of exertional dyspnea, nonproductive cough, and wheezing. What are these manifestations signs of in the lung transplant patient? a) Pulmonary infarction b) Pulmonary hypertension c) Cytomegalovirus (CMV) d) Bronchiolitis obliterans (BOS)

d) Bronchiolitis obliterans (BOS) Bronchiolitis obliterans (BOS) is a manifestation of chronic rejection and is characterized by airflow obstruction progressing over time with a gradual onset of exertional dyspnea, nonproductive cough, wheezing, and/or low-grade fever. Pulmonary infarction occurs with lack of blood flow to the bronchial tissue or preexisting lung disease. With pulmonary hypertension, the pulmonary pressures are elevated and can be idiopathic or secondarily due to parenchymal lung disease that causes anatomic or vascular changes leading to pulmonary hypertension. CMV pneumonia is the most common opportunistic infection 1 to 4 months after lung transplant.

What is the priority nursing intervention in helping a patient expectorate thick lung secretions? a) Humidify the oxygen as able. b) Administer cough suppressant q4hr. c) Teach patient to splint the affected area. d) Increase fluid intake to 3 L/day if tolerated.

d) Increase fluid intake to 3 L/day if tolerated. Although several interventions may help the patient expectorate mucus, the highest priority should be on increasing fluid intake, which will liquefy the secretions so that the patient can expectorate them more easily. Humidifying the oxygen is also helpful but is not the primary intervention. Teaching the patient to splint the affected area may also be helpful in decreasing discomfort but does not assist in expectoration of thick secretions.

The patient had video-assisted thoracic surgery (VATS) to perform a lobectomy. What does the nurse know is the reason for using this type of surgery? a) The patient has lung cancer. b) The incision will be medial sternal or lateral. c) Chest tubes will not be needed postoperatively. d) Less discomfort and faster return to normal activity

d) Less discomfort and faster return to normal activity The VATS procedure uses minimally invasive incisions that cause less discomfort and allow faster healing and return to normal activity as well as lower morbidity risk and fewer complications. Many surgeries can be done for lung cancer, but pneumonectomy via thoracotomy is the most common surgery for lung cancer. The incision for a thoracotomy is commonly a medial sternotomy or a lateral approach. A chest tube will be needed postoperatively for VATS.

A 71-year-old patient is admitted with acute respiratory distress related to cor pulmonale. Which nursing intervention is most appropriate during admission of this patient? a) Perform a comprehensive health history with the patient to review prior respiratory problems. b) Complete a full physical examination to determine the effect of the respiratory distress on other body functions. c) Delay any physical assessment of the patient and review with the family the patient's history of respiratory problems. d) Perform a physical assessment of the respiratory system and ask specific questions related to this episode of respiratory distress.

d) Perform a physical assessment of the respiratory system and ask specific questions related to this episode of respiratory distress. Because the patient is having respiratory difficulty, the nurse should ask specific questions about this episode and perform a physical assessment of this system. Further history taking and physical examination of other body systems can proceed once the patient's acute respiratory distress is being managed.

The nurse is caring for a 73-year-old patient who underwent a left total knee arthroplasty. On the third postoperative day, the patient complains of shortness of breath, slight chest pain, and that "something is wrong." Temperature is 98.4° F, blood pressure 130/88, respirations 36/minute, and oxygen saturation 91% on room air. What action should the nurse take first? a) Notify the physician. b) Administer a nitroglycerin tablet sublingually. c) Conduct a thorough assessment of the chest pain. d) Sit the patient up in bed as tolerated and apply oxygen.

d) Sit the patient up in bed as tolerated and apply oxygen. The patient's clinical picture is most likely pulmonary embolus, and the first action the nurse takes should be to assist with the patient's respirations. For this reason, the nurse should sit the patient up as tolerated and apply oxygen before notifying the physician. The nitroglycerin tablet would not be helpful, and the oxygenation status is a bigger problem than the slight chest pain at this time.

An appropriate nursing intervention for a patient with pneumonia with the nursing diagnosis of ineffective airway clearance related to thick secretions and fatigue would be to: a. perform postural drainage every hour b. provide analgesics as ordered to promote patient comfort c. administer O2 as prescribed to maintain optimal oxygen levels d. teach the patient how to cough effectively to bring secretions to the mouth.

d. teach the patient how to cough effectively to bring secretions to the mouth.


Related study sets

Top 10 Most Spoken Languages in the world (and 10 of the Least Spoken)

View Set

Entrepreneurial Small Business Chapter 14 Small Business Finance: Using Equity, Debt and Gifts

View Set

google adwords knowledge check questions

View Set

Musculoskeletal System (Med/Surg)

View Set

Modules 8 - 10: ACLs and Firewalls Group Exam

View Set

¡Inténtalo!: Subject Pronouns & Present Forms of 'ser'

View Set

Marketing Management Final Review

View Set

Med Surg Chapter 48: Management of Patients With Intestinal and Rectal Disorders

View Set

"Repetition, parallelism, alliteration and antithesis".(For ESOL Students)

View Set